Breast Reconstruction Flashcards
A 39-year €‘old woman is referred for consultation regarding reconstruction of the left breast two years after mastectomy for breast cancer. Implantation of a silicone prosthesis and reconstruction with a pedicled latissimus dorsi musculocutaneous flap are planned. This patient will most likely show deficits in which of the following shoulder functions?
(A) Extension and abduction
(B) Extension and adduction
(C) Flexion and abduction
(D) Flexion and adduction
(E) No deficits
The correct response is Option B.
Transfer of the latissimus dorsi musculocutaneous flap is associated with deficits in extension and adduction.
The latissimus muscle acts synergistically with six other muscles in the shoulder. The primary contribution of the muscle is in shoulder extension, adduction, and medial rotation. There are definite biomechanical changes that occur in the shoulder girdle with latissimus muscle transfer. The synergistic action of the teres major muscle compensates for loss of the latissimus muscle. This leads to teres major hypertrophy in the long term. A study in 1986 evaluated a series of patients with latissimus muscle transfer and found a decrease in total shoulder strength up to 34% compared to the contralateral side. When tested specifically for the latissimus muscle function, the average weakness was 18%. Active range of motion at the shoulder with extension and adduction was decreased by 5% and 0%, respectively.
In 1992, one study used isometric, isotonic, and isokinetic strength tests for evaluation after latissimus transfer. The authors concluded that forced extension is weaker only when the arms are in 60 degrees of flexion. There was no loss of range of motion. Another study done in 1995 reexamined change in muscle power and endurance after latissimus transfer. The authors concluded that women who underwent unilateral pedicled latissimus transfer showed a deficit of power and endurance in shoulder extension and adduction. This was seen in three specific work activities: ladder climbing, overhead painting, and pushing up from a chair. A more rapid onset of fatigue during prolonged activities involving these motions, including swimming, is noted.
A 16-year-old girl is brought to the office by her parents because she has had worsening pain in the right breast (shown) for the past eight months. Ten years ago, she sustained a full-thickness burn to the back, right chest, and abdomen requiring excision and split-thickness skin grafting. Physical examination shows contracture of the scar and lateral expansion of the breast. After release of the burn scar contractures, a defect of 200 cm2 is created. Which of the following is most appropriate for coverage of this defect?
(A) Dermal regeneration template (Integra)
(B) Latissimus dorsi musculocutaneous flap
(C) Meshed split-thickness skin graft
(D) Nonmeshed thin split-thickness skin graft
(E) Z-plasty release of the constricted scar
The correct response is Option A.
The patient described is suffering from pain caused by a developing breast that is constricted under a nonexpanding split-thickness skin graft placed at the time of the burn excision. The burn scar contractures are first released to allow expansion of the underlying breast. The decision for coverage material is then based on what will most likely permit further expansion of the breast.
Dermal regeneration template (Integra) has been shown to allow secondary expansion and is the most appropriate choice for the scenario described. The wound is covered with an expandable dermal layer, which is then covered again by a split-thickness skin graft, leaving a minimal donor scar.
A Z-plasty would be inadequate to release this large scar.
Full-thickness skin grafting would allow greater breast expansion and less secondary burn scar contracture, but a graft of the size needed would be difficult to obtain.
A latissimus dorsi flap would not be the best option for the patient described because she has a history of burns on the back. Also, a latissimus dorsi flap would be bulky, and the patient described does not require additional volume.
Nonmeshed thin split-thickness skin grafting would give a superior aesthetic result compared with meshed split-thickness skin grafting; however, both would result in secondary burn scar contracture, which would not allow further expansion of the breast.
A 52-year-old woman with breast cancer undergoes right mastectomy and reconstruction with a free transverse rectus abdominis musculocutaneous (TRAM) flap. The procedure is uneventful. In the recovery room, the patient’s husband says that she has been smoking one pack of cigarettes daily up to the day of surgery. Which of the following postoperative complications is most likely to occur?
A) Flap hematoma
B) Mastectomy skin loss
C) Microvascular thrombosis
D) Partial flap loss
E) Superficial infection
The correct response is Option B.
Patients who smoke cigarettes and who undergo breast reconstruction with a free flap have a higher rate of mastectomy skin loss. In addition, they have a high rate of donor site abdominal flap necrosis, umbilical necrosis, and hernias. There is no increase in microsurgical complications, flap-related complications (partial flap loss or fat necrosis), infections, or hematomas.
Current recommendations are for patients to stop smoking at least 4 weeks prior to breast reconstruction. Patients who quit smoking prior to this period have a lower risk of perioperative complications when compared to active smokers.
A 43-year-old woman undergoes the second stage of tissue expander–based breast reconstruction. Exchange of the tissue expander for a smooth round silicone implant is planned along with a superior capsulotomy and fat grafting to the upper pole for contour improvement. Which of the following is an increased risk associated with fat grafting to the breast in this patient?
A) Anaplastic large cell lymphoma
B) Benign lesions
C) Hypopigmentation
D) Infection
E) Recurrent breast cancer
The correct response is Option B.
Autologous fat grafting is a widely accepted technique in breast reconstruction. A large systematic review recently confirmed the oncologic safety of this technique but did report a significant incidence of benign lesions including cysts and calcifications. Fat grafting is not associated with an increased risk of recurrent breast cancer, infection or hypopigmentation.
A 41-year-old man is referred to the office because of a mass on his left breast that has been growing rapidly for 1 month. Examination of the specimen obtained on core needle biopsy confirms invasive mammary carcinoma. Genetic testing results are positive for BRCA2. In addition to an increased risk of male breast cancer, which of the following other types of cancer is most likely to be associated with this patient?
A) Colon
B) Lung
C) Pancreatic
D) Renal
E) Thyroid
The correct response is Option C.
Men with harmful BRCA1 mutations also have an increased risk of breast cancer and, possibly, pancreatic cancer, testicular cancer, and early onset prostate cancer. However, male breast cancer, pancreatic cancer, and prostate cancer appear to be more strongly associated with BRCA2 gene mutations. Colon, lung, renal, and thyroid cancers are not associated with the BRCA1 and BRCA2 gene mutations.
A 55-year old woman undergoes modified radical mastectomy with immediate first-stage reconstruction of the right breast with a tissue expander. Before the second stage to exchange the tissue expander with a permanent prosthesis is initiated, pathology results from analysis of tissue from the right breast indicate metastatic carcinoma of four axillary lymph nodes. Radiation therapy is recommended. Which is the most optimal choice for how the reconstruction should proceed?
A) Complete the tissue expansion before radiation and exchange for an implant after radiation
B) Deflate the tissue expander before radiation and reinflate the tissue expander and exchange with an implant after radiation
C) Remove the tissue expander and reconstruct the breast with an autologous flap before radiation
D) Remove the tissue expander before radiation; after radiation, reinsert and expand the tissue expander and then exchange for an implant
E) Maintain the tissue expander in place during the radiation. Replace the expander with an autologous flap after radiation
Correct answer is option E.
Breast reconstruction after radiation therapy is best accomplished with an autologous flap. Doing so recruits fresh, well-vascularized, non-irradiated tissue to the irradiated chest wall. Many authors have removed complication rates of up to 50% when an implant is used (without an autologous flap) for breast reconstruction after radiation therapy. In the setting of radiation, if an implant is used without a flap, there is an increased risk of infection and exposure of the prosthesis, leading to failure of the breast reconstruction. Radiation also increases the risk for capsular contracture, resulting in a poor aesthetic result.
Most surgeons recommend waiting at least 6 months from the completion of radiation before performing the second stage of reconstruction. This period of time allows for the acute inflammatory effects of radiation to dissipate. This is especially important for wound healing and – if a microsurgical flap is being used for the reconstruction – to ensure an optimal environment for microvascular anastomosis to the internal mammary vessels.
It is safe to leave the tissue expander in place during radiation. Many surgeons purposefully approach breast reconstruction in this manner; it is termed delayed-immediate reconstruction. When radiation is anticipated, placing a tissue expander at the time of the mastectomy allows for a larger amount of native mastectomy skin flap to be maintained, which many plastic surgeons believe contributes to a superior cosmetic outcome. A delayed breast reconstruction after radiation therapy (and without a tissue expander) is possible, but doing so requires a greater amount of pale abdominal skin for the breast reconstruction, which is thought to not be aesthetically ideal. One potential disadvantage of the delayed immediate approach is the increased infection risk it confers compared to not placing a prosthetic device. However this risk is not thought to be prohibitively high.
A 35-year-old woman comes to the office to discuss a recent diagnosis of breast cancer. Recent mammography showed diffuse microcalcifications throughout the breast, and needle biopsy showed infiltrating ductal carcinoma. On physical examination, some retraction of the skin in the lower outer quadrant of the breast is noted. She wears a size 36C brassiere. The patient reports that she is currently considering whether to have lumpectomy and radiation therapy or mastectomy. Which of the following features of this clinical scenario is a CONTRAINDICATION to breast conservation therapy?
A) Breast size
B) Mammographic findings
C) Patient age
D) Skin retraction on physical examination of the breast
E) Tumor pathology
The correct response is Option B.
Breast conservation therapy (BCT) refers to breast conserving therapy followed by moderate-dose radiation to eradicate microscopic residual disease. The goal is to provide the equivalent survival of mastectomy while maintaining a cosmetically acceptable appearance with a low rate of recurrence.
When considering breast conservation therapy or mastectomy, the needs and desires of each patient should be addressed. Age alone is not a contraindication to BCT, but overall health and comorbidities should be considered. Histologic subtype and pathology are not contraindications to BCT as long as the tumor is not diffuse and can be safely excised with negative margins. Similarly, breast size needs to be considered along with tumor size and location, but “small” or “large” breasts are not indications or contraindications. While retraction of the skin, nipple, or breast parenchyma is not an absolute contraindication to BCT, as long as negative margins can be safely removed, the cosmetic impact of their involvement should be considered.
There are few absolute contraindications to BCT, but they include:
Multicentric disease with two or more tumors in separate quadrants of the breast such that they cannot be encompassed in a single excision
Diffuse malignant microcalcifications on mammography
A history of prior radiation in the same breast or chest wall
Pregnancy
Persistently positive margins despite re-excision
A 53-year-old woman is evaluated for left-sided nipple reconstruction after mastectomy. She has scars on the left breast from a previous breast biopsy, as well as from the mastectomy itself. Nipple reconstruction must be designed around the scars. In single-pedicle nipple reconstruction, which of the following provides the blood supply to the pedicle?
A) Internal mammary artery perforators
B) Posterior intercostal arteries
C) Subdermal plexus
D) Superior intercostal artery
E) Thoracoacromial artery perforators
The correct response is Option C.
Single-pedicle nipple reconstructions, which include such techniques as the skate flap, star flap, C-V flap, and opposing tab flaps as well as other variations, create nipples from remaining mastectomy skin through adjacent tissue transfer. The flap derives its blood supply from the subdermal plexus.
The creation of the flap must keep this blood supply in mind. The flap design must avoid previous scars at the flap base and must integrate the subcutaneous fat at the base of the pedicle.
The internal mammary artery supplies the breast itself and the nipple-areola complex, and the thoracoacromial artery supplies the pectoralis muscle and the breast. The posterior intercostal arteries supply the intercostal spaces. The superior intercostal artery arises from the costocervical trunk, off of the subclavian artery, and supplies the intercostal spaces.
A 48-year-old woman undergoes skin-sparing mastectomy followed by immediate breast reconstruction with implantation of subpectoral prostheses. Which of the following best describes the resulting effect on pectoralis torque strength in this patient?
A ) Decreased on the operated side
B ) Decreased on the side of hand dominance
C ) Increased on the operated side
D ) Increased on the side of hand dominance
E ) No effect
The correct response is Option A.
Much attention has been given to abdominal wall function and complications following autologous breast reconstruction utilizing abdominal wall tissue in the form of transverse rectus abdominis musculocutaneous (TRAM), free TRAM, muscle-sparing TRAM, and deep inferior epigastric perforator free tissue transfers.
De Haan and colleagues examined the effect of immediate subpectoral prosthetic breast reconstruction following skin-sparing mastectomy on pectoralis function by measuring bilateral isometric arm strength in patients undergoing unilateral breast reconstruction. After correcting for the effect of hand dominance, the authors found a statistically significant decrease in torque strength on the operated side by 20.1% compared to the nonoperated side. This loss represents approximately half of the torque needed by healthy subjects over 50 years of age to rise from a chair and was therefore considered substantial.
A 60-year-old woman receives low-molecular-weight heparin (LMWH) 40 U subcutaneously 1 hour before undergoing breast reconstruction using a unilateral transverse rectus abdominis musculocutaneous (TRAM) flap. Weight is 185 lb (84 kg); BMI is 32 kg/m2. Which of the following is the most likely effect of the LMWH on perioperative risks in this patient?
A ) Decreased risk of flap failure
B ) Decreased risk of postoperative hematoma
C ) Decreased risk of pulmonary thromboembolism
D ) Increased risk of blood transfusion
E ) Increased risk of infection
The correct response is Option C.
There are no uniform standards or guidelines for the routine use of chemoprophylaxis of venous thromboembolism in plastic surgery. At a minimum, lower extremity mechanical compression devices should be used on all patients undergoing general anesthesia. The patient described has multiple risk factors for developing deep venous thrombosis (DVT) and pulmonary embolism, such as an age over 50, prolonged surgery time, malignancy, and being overweight.
In such high-risk patients, data support the use of chemoprophylaxis with either standard heparin or low-molecular-weight heparin given subcutaneously. As the risk of developing DVT begins with anesthesia induction, it is generally recommended that heparin therapy be started before surgery. Out of concern for bleeding, some surgeons start heparin therapy in the early postoperative period. While more data are needed to clarify the optimal start time of therapy, it is clear that starting therapy before surgical incision is generally safe when dosed appropriately.
In large retrospective studies, chemoprophylaxis patients were not more likely to require blood transfusion, though they do demonstrate a slightly greater decrease in postoperative hemoglobin as compared with control patients. There is clearly a decreased risk of postoperative DVT and pulmonary embolism, both clinically apparent and asymptomatic. The data regarding postoperative hematoma are less clear, with some studies showing no increase in €œtake-backs € to the operating room for hematoma with chemoprophylaxis. One study did show an increase in hematomas in oncologic breast surgery with LMWH versus standard, unfractionated heparin. There is no association between flap survival or infection and subcutaneous heparin use.
Until reliable prospective, randomized data of sufficient study size are available to demonstrate optimal treatment, the studies seem to support the routine use of mechanical and perioperative subcutaneous heparin prophylaxis in high-risk patients.
The capsules from patients with breast implant–associated anaplastic large-cell lymphoma (ALCL) have significant presence of which of the following bacteria?
A) Escherichia coli
B) Ralstonia pickettii
C) Staphylococcus aureus
D) Pseudomonas aeruginosa
E) Serratia marcescens
The correct response is Option B.
Most concerning in the past two decades is the incidence of breast implant–associated anaplastic large-cell lymphoma (ALCL). This entity was first diagnosed and associated with breast implants in 1997, and is almost only associated with a history of textured implants and/or tissue expanders. The most common presentation of these patients is late seroma, with some patients presenting with mass, tumor erosion, or lymph node metastasis. A recent review of the world literature on this entity include the following: (1) 173 cases were documented, (2) no cases were found in patients with documented smooth devices only (although this remains controversial, as the data in many cases are incomplete), (3) there may be an associated genetic predisposition as suggested for cutaneous T-cell lymphoma, and (4) the cause is likely multifactorial.
Bacterial biofilm is thought to be an inciting factor for the development of both breast-implant related ALCL and Non-Tumor related capsule contractures. The capsules from patients with tumor had significant presence of Gram-negative bacteria (Ralstonia species) compared to nontumor capsules (Staphylococcus species). Such data may support the bacterial induction model, as there are also other types of implant-associated lymphomas.
Two years after undergoing modified radical mastectomy on the right, a 36-year-old woman desires reconstruction with an implant. Physical examination shows minimal bulk in the inferior and lateral portions of the right pectoralis major muscle.
The most likely cause is denervation of which of the following nerves during mastectomy?
(A) Fourth intercostal
(B) Lateral pectoral
(C) Long thoracic
(D) Medial pectoral
(E) Thoracodorsal
The correct response is Option D.
The minimal bulk in the inferior and lateral portions of the right pectoralis major muscle in this patient most likely results from denervation of the medial pectoral nerve, which is injured often during mastectomy. The pectoralis major is innervated by the medial and lateral pectoral nerves, named for their respective cords of origin from the brachial plexus. The lateral nerve, which arises from C5-6, actually supplies the medial portion of the pectoralis major muscle, and the medial nerve, derived from C8-T1, supplies the lower and lateral sternal portion of the pectoralis major muscle and the pectoralis minor muscle.
Because injury to the medial pectoral nerve results in severe atrophy of the lower half of the muscle, implant coverage will be sparse in this region. It is important for the plastic surgeon to assess the status of the pectoralis muscle before attempting implant augmentation; this can be accomplished by having the patient place her hands on her hips and contract her chest muscles.
The fourth intercostal nerve supplies sensation to the nipple-areola complex. The long thoracic nerve, commonly referred to as the long thoracic nerve of Bell, supplies motor innervation to the serratus muscle, and injury may result in winging of the scapula. The thoracodorsal nerve provides innervation to the latissimus dorsi muscle.
An otherwise healthy 54-year-old perimenopausal woman is scheduled for a mastectomy for biopsy-proven right-sided grade 2 ductal carcinoma. According to the National Comprehensive Cancer Network (NCCN) guidelines, postmastectomy radiation therapy will be the standard of care for this patient if she has which of the following surgical outcomes?
A) 1-cm surgical margins, four positive axillary lymph nodes
B) 1-cm surgical margins, one positive axillary sentinel node
C) 1-mm surgical margins, no positive axillary nodes
D) 5-mm surgical margins, no positive axillary nodes
E) 5-mm surgical margins, three positive axillary nodes
The correct response is Option A.
Traditionally, the need for radiation therapy has been a contraindication for implant-based reconstruction, and autologous reconstruction is the conservative gold standard for women with advanced cancer needing postmastectomy radiation. More recently, there have been reports of successful implant based reconstruction in the setting of postmastectomy radiation that have similar complication profiles and good oncologic outcomes compared with autologous reconstruction. Protocols vary between those that radiate the expander and then expand, and those that expand and then radiate the permanent implant. Being able to anticipate which patient will require postmastectomy radiation is essential for joint decision making about breast reconstruction with the patient prior to her mastectomy.
By National Comprehensive Cancer Network (NCCN) guidelines, relative indications for postmastectomy radiation therapy include: positive sentinel node with unknown status of other axillary nodes, one to three positive nodes on permanent histology, and close surgical margins (less than 5 mm). Postmastectomy radiation is recommended as the standard of care in the situations of positive surgical margins with the inability to get clear margins and four or more positive lymph nodes.
An otherwise healthy, 45-year-old woman presents for breast reconstruction. She underwent a mastectomy 1 year ago with no immediate reconstruction followed by post-mastectomy radiation therapy. She does not want anything performed to the contralateral breast and does not want to have an implant. She has a history of an abdominoplasty. The plastic surgeon plans to perform stacked profunda artery perforator flaps for the unilateral reconstruction. Which of the following is the most appropriate option for the recipient vessels?
A) Ipsilateral and contralateral antegrade internal mammary vessels
B) Ipsilateral antegrade and retrograde internal mammary vessels
C) Ipsilateral antegrade and retrograde thoracodorsal vessels
D) Ipsilateral antegrade internal mammary and thoracoacromial vessels
E) Ipsilateral antegrade thoracodorsal and thoracoacromial vessels
The correct response is Option B.
It has been demonstrated that the retrograde intermammary vessels are a viable and reliable choice for stacked flap reconstruction. The benefit of using these as the recipient vessels is that it does not add another recipient site to the operation.
Ipsilateral and contralateral antegrade internal mammary vessels are incorrect because the patient does not want to have any procedures performed on her contralateral breast, although this has been described as an option for autologous breast reconstruction.
The thoracodorsal vessels, prior to development of the internal mammary vessels as the preferred option for autologous breast reconstruction, were commonly used for recipients, but utilizing these can negate the option of a latissimus dorsi as a salvage procedure in the setting of flap failure, especially if the plastic surgeon were to use the antegrade and retrograde vessels. Ipsilateral antegrade internal mammary and thoracodorsal vessels would be correct as well, but it was not given as an option. Thoracoacromial vessels are not typically used in autologous breast reconstruction, aside from being a lifeboat for venous outflow.
A 50-year-old female with a BMI of 37 has a 7 cm invasive ductal carcinoma and undergoes mastectomy and immediate reconstruction with a free deep inferior epigastric perforator flap. Post mastectomy radiation therapy is recommended for her patient due to her tumor size, but citing concerns about the potential negative impact of the radiotherapy on her flap, she declines treatment. 3 months later she is treated with fat grafting to the superior pole, in order to provide a more gradual and anatomic transition between the chest wall and breast. Four years after her mastectomy she presents to your office with a palpable lump within her right DIEP flap. The lump is biopsied and demonstrates a recurrent invasive ductal carcinoma. Which factor is most likely to have contributed to this patient’s cancer recurrence?
A) The fact she received a DIEP flap for her reconstruction
B) The fact that she received fat grafting during her reconstruction revision
C) She did not receive radiotherapy
D) Her elevated BMI
Correct answer is option C.
Fat grafting has been demonstrated to be oncologically safe in several clinical studies. To-date, there is no evidence that receiving fat grafting contributes to an elevated risk for cancer recurrence. Furthermore, there is no evidence that receiving breast reconstruction – including any technique – is associated with an increased risk for cancer recurrence. The most likely reason this woman developed a cancer recurrence is the fact that she chose to ignore medical advice and not receive radiotherapy, despite a clear indication for it.
A 48-year-old woman undergoes immediate unilateral breast reconstruction with a free deep inferior epigastric artery perforator (DIEP) flap. At the conclusion of the procedure, the flap skin paddle is noted to have venous congestion. Upon reexploration, the venous anastomosis appears patent with venous outflow detected by handheld pencil Doppler evaluation, but the flap continues to have venous congestion with brisk capillary refill. Which of the following is the most appropriate next step in management?
A) Apply leeches postoperatively
B) Loosely re-inset the flap and monitor closely
C) Perform a second venous anastomosis using the superficial inferior epigastric vein
D) Perform a second venous anastomosis using the vena comitans
E) Revise the venous anastomosis using a hand-sewn technique
The correct response is Option C.
Preservation of the superficial inferior epigastric veins (SIEV) during flap harvest is a useful preventive measure in microsurgical free tissue transfer operations. These veins can serve as important lifeboats to augment venous outflow in the setting of venous congestion. Typically, if a free flap demonstrates venous congestion, the inset should be taken down and the pedicle, recipient vessels, and anastomoses should be interrogated. Simple issues, such as mechanical compression or twisting of the vein, should be ruled out. Next, the SIEV should be inspected. If engorged, the flap is likely reliant on superficial outflow, and this vein should be connected to a recipient vessel to augment the venous outflow of the flap. Options for recipient veins include an anterograde branch on the pedicle vena comitans, or in a retrograde fashion to the vena comitans that was not used in the initial set of anastomoses.
In this case scenario, the flap continued to demonstrate venous congestion intraoperatively. This makes it unlikely that tension or pressure from the inset of the flap was causing the venous outflow obstruction. Furthermore, leech therapy is not indicated for a free flap with global venous congestion.
The venous coupling device is safe and effective for the anastomosis of veins in DIEP flap surgery. It has not been associated with patency rates that are different from hand-sewn anastomoses. The coupling device, however, has been shown to reduce the microsurgery time.
The use of one or two veins in microsurgical free tissue transfer is a topic that has been debated for several years. While some studies indicate that the use of two venous connections may reduce the velocity of blood flow across the anastomosis, there is not sufficient data to support differences in flap outcomes or thrombotic events. Therefore, the routine use of a second vein is largely up to surgeon preference.
A 33-year-old woman comes to the office with a 6-cm rapidly growing tumor of the left breast. She wears a size 36C brassiere. The tumor has a bluish hue and skin veins are dilated. A phyllodes tumor is diagnosed, and surgical excision is planned. Which of the following is the most appropriate surgical procedure to treat this patient?
A) Excision with 1-cm margin
B) Excision with 2-cm margin
C) Excisional biopsy
D) Modified radical mastectomy
E) Radical mastectomy
The correct response is Option A.
Phyllodes are large benign tumors that occur primarily in the perimenopausal patient. Previously, they were referred to as cystosarcoma phyllodes, a term coined in 1838 because the tumors are fleshy and have a gross leaf-like intracanalicular growth pattern. However, this is a misnomer because these tumors do not behave like sarcomas and are rarely malignant. The histologic characteristics that separate fibroadenomas from phyllodes tumors are not well defined and have been somewhat controversial. Nevertheless, phyllodes tumors typically are large fibroadenomas that histologically have more stromal cellularity than that seen in the typical fibroadenoma. The classification of benign versus malignant phyllodes tumors is not sharply delineated, and the term borderline lesion may be more appropriate. Borderline lesions have more mitoses per high-power field and moderate nuclear pleomorphism. They have a tendency to recur after local excision but do not demonstrate true malignant behavior. When metastases of a phyllodes tumor have been reported, there have been obvious sarcomatous elements such as liposarcoma or rhabdomyosarcoma in the lesion.
The surgical treatment of phyllodes tumors has recently been redefined. In the past, simple or radical mastectomies were recommended for the treatment of large phyllodes tumors. Currently, most surgeons perform more conservative surgery. Several clinical studies have recommended the excision of tumors with 1-cm clear margins or mastectomy if breast conservation is impossible.
A 45-year-old woman undergoes bilateral nipple-sparing mastectomy with immediate tissue expander reconstruction for T3N2 breast cancer. She completes radiotherapy and resumes expansion in the office 4 months after surgery. Her husband calls the office noting that the patient is reporting headache and requests additional pain medication when she suddenly demonstrates seizure activity. She is taken to the emergency department for immediate evaluation and stabilization. Which of the following imaging modalities should be used with caution for this patient?
A) CT scan
B) Fluoroscopy
C) MRI
D) Positron emission study
E) Ultrasound
The correct response is Option C.
The majority of breast tissue expanders include a ferromagnetic port and are currently labeled as MRI-unsafe due to the potential interaction with the magnetic field of the machine. There are reports of several MRI-related complications: malposition, pain or burning sensation, polarity reversal, port dislodgement, and thermal injuries. However, several subsequent studies have demonstrated modifications to minimize risks in patients requiring diagnostic MRI of either the contralateral breast, perforator mapping, or evaluation of brain/spine metastases. This patient may need MRI evaluation of the brain to diagnose metastatic disease of the brain. While some imaging technicians may be unwilling to perform MRI due to the warnings, studies suggest use of a 1.5T MRI, saline filling, and prone positioning are effective strategies to minimize risks and safely perform MRI rather than explant the device(s).
CT scanning can be safely performed with no contraindication due to expanders. Positron emission studies also do not interfere with expanders. Ultrasound is frequently used to safely assess and treat possible fluid collections or infections associated with expanders. Despite her previous radiation therapy, fluoroscopy can still be safely performed while taking care to minimize radiation exposure to critical structures.
A 49-year-old woman who underwent first-stage left breast reconstruction with a tissue expander presents for follow-up to discuss expander to implant exchange. The reconstructed side has more volume and superior fullness compared with the native breast, and the patient prefers the reconstructed side. A photograph is shown. In addition to implant exchange, which of the following would most likely give the patient the greatest satisfaction regarding the appearance of her breasts?
A) Left breast fat grafting
B) Left breast flap reconstruction
C) Right breast implant augmentation
D) Right breast mastopexy
E) No additional procedures
The correct response is Option C.
The patient shown in the initial photograph complains of lack of symmetry between the reconstructed breast and the native, contralateral side. The volume and superior fullness in the reconstructed breast are a result of an implant-based reconstruction. Studies have shown improved patient satisfaction with breast reconstruction when contralateral augmentation is performed for symmetry. For this patient who was happy with an implant-based reconstruction, symmetry was best achieved with a contralateral breast augmentation. In the additional image shown, she is pictured after tissue expander to implant exchange, nipple-areola complex reconstruction, and right breast augmentation. The other choices are less ideal and would not necessarily give the patient the symmetry or satisfaction she desires.
A 36-year-old woman desires breast reconstruction one year after undergoing right modified radical mastectomy followed by radiation therapy. She is 5 ft 4 in tall, weighs 135 lb, and is otherwise healthy. The left cup size of her bra is 32B.
Which of the following is the most appropriate reconstructive option in this patient?
(A) Reconstruction with a latissimus dorsi flap and a saline-filled implant
(B) Reconstruction with a TRAM flap
(C) One-stage reconstruction with a silicone gel-filled implant
(D) Two-stage reconstruction with a tissue expander and a saline-filled implant
(E) Two-stage reconstruction with a tissue expander and a silicone gel-filled implant
The correct response is Option B.
Reconstruction with a TRAM flap is the most appropriate choice for this 36-year-old woman. Transfer of autologous tissue alone is the best method in any patient who has previously undergone radiation therapy. Use of an implant, whether saline- or silicone gel-filled, is associated with a significant increase in capsular contracture and other complications in previously irradiated patients. The latissimus dorsi flap is reserved for those patients who are not good candidates for TRAM flap reconstruction or as a salvage technique following periprosthetic contracture in patients who underwent implant reconstruction following radiation therapy.
A 42-year-old woman with a 3-cm invasive ductal carcinoma of the right breast is evaluated for breast reconstruction. She has not decided how she wants to manage her contralateral breast. Regarding eliciting a family history, which of the following cancers is associated with a mutation in a breast cancer-susceptibility gene?
A) Colon
B) Esophageal
C) Lung
D) Pancreatic
E) Thyroid
The correct response is Option D.
The breast cancer-susceptibility gene types 1 and 2 (BRCA1 and BRCA2) are tumor suppressor genes. Mutations in BRCA1and BRCA2 are associated with hereditary breast and ovarian cancers. Additionally, they can be associated with increased risks of pancreatic and prostate cancer. Thyroid, lung, esophageal, and colon cancer are not associated with increased risks of BRCA1 and BRCA2 mutations.
Which of the following legislative acts mandates insurance coverage of breast reconstruction after total mastectomy as well as coverage of any associated symmetry procedures for the contralateral breast?
A) Affordable Care Act
B) Breast Cancer Patient Education Act
C) SB-255 Amendment to the Knox-Keene Health Care Service Plan Act
D) Women’s Health and Cancer Rights Act
The correct response is Option D.
The Women’s Health and Cancer Rights Act of 1998 (WHCRA) is a federal law that mandates the coverage of breast reconstruction after mastectomy as well as coverage of any associated symmetry procedures for the contralateral breast. Although this federal law was enacted more than 20 years ago, there are still significant disparities in access to breast reconstruction and a lack of education regarding the options available for breast reconstruction.
The Breast Cancer Patient Education Act of 2015 is a federal law that requires the Secretary of Health and Human Services to implement an educational campaign to inform breast cancer patients about access, availability, and options for breast reconstruction after mastectomy. SB-255 is an amendment in the state of California to the Knox-Keene Health Care Service Plan Act that includes “lumpectomy” for treatment of breast cancer in the definition of “mastectomy” and mandates access to insurance coverage of breast reconstruction after lumpectomy in the state of California. It is important to note that the WHCRA only mandates coverage of breast reconstruction after mastectomy, not after breast conservation therapies such as lumpectomy. In fact, there is no current legislation mandating insurance coverage for all types of breast reconstruction, including breast reduction, mastopexy, or implant complications after aesthetic surgeries.
The Affordable Care Act (ACA) is a comprehensive health care reform law enacted in March of 2010. The three primary goals of the ACA are to make affordable health insurance available to more people, to expand the Medicaid program to cover all adults with income below 138% of the federal poverty level, and to support innovative medical care delivery methods designed to lower the costs of health care generally. There is nothing specifically in reference to breast cancer or breast reconstruction care within the ACA.
An 18-year-old man with Klinefelter syndrome is referred to the office because of overdeveloped breasts. Diagnostic workup testing is ordered. The likelihood of breast cancer is increased if testing shows which of the following results?
A ) Decreased level of plasma follicle-stimulating hormone
B ) Decreased level of plasma luteinizing hormone
C ) High number of progesterone receptors in the mesenchymal breast tissue
D ) Increased level of plasma testosterone
E ) Low number of estrogen receptors in the mesenchymal breast tissue
The correct response is Option C.
Klinefelter syndrome is a relatively rare disorder with an incidence of approximately 1:100,000, yet it represents a very important subset of patients with gynecomastia. Approximately one half of patients with typical Klinefelter syndrome (47,XXY) and one third of those with the mosaic variety (46,XY/47,XXY) develop gynecomastia. Those exhibiting gynecomastia have elevated plasma follicle-stimulating and luteinizing hormone levels, along with decreased plasma testosterone levels, suggesting a hormonal link with gynecomastia in the Klinefelter population.
The presence of estrogen and progesterone receptors in elevated concentration in patients with Klinefelter syndrome provides a potential mechanism by which these patients develop breast neoplasms. The absence of elevated estrogen and progesterone receptors in patients with idiopathic gynecomastia might help to clarify why these patients rarely develop breast malignancies.
Gynecomastia has many identifiable causes, although most cases are idiopathic. Recent studies have shown strong evidence for the estrogen-stimulating effects of breast tissue development and support for an inhibitory androgenic effect. Decreases in the androgen-to-estrogen ratio have also been associated with development of gynecomastia. No clear etiologic classification has been suggested based solely on hormonal influences. Instead, most physicians accept an arbitrary classification based on physiologic, pathologic, pharmacologic, and idiopathic causes. Idiopathic causes (25%) are the most common.
A 35-year-old woman with a Stage T2 infiltrating ductal carcinoma is scheduled to undergo a skin-sparing, right total mastectomy and a nipple-sparing, left prophylactic mastectomy. The possibility of adjuvant radiation therapy to the right breast depends on the final surgical pathology. The patient has a history of smoking. BMI is 28 kg/m2. She wears a brassiere with a D cup and would like the postoperative result to be of a similar size. Which of the following immediate bilateral reconstructive techniques is most appropriate for this patient?
A) Abdominal-based free flaps
B) Gluteal-based free flaps
C) Latissimus dorsi myocutaneous flaps and silicone implants
D) Silicone implants and acellular dermal matrix
E) Tissue expanders and acellular dermal matrix
The correct response is Option E.
For this patient in whom postoperative radiation therapy is possible, the best first-stage, immediate reconstructive approach is placement of tissue expanders with acellular dermal matrix. The outcome of immediate autologous flap reconstruction may be compromised if subjected to adjuvant radiation therapy and is best delayed until after such treatment has been rendered. Although successful, cost-effective outcomes are possible with a single-stage, direct-to-implant approach, this patient has risk factors for early revision and implant failure due to her large breasts and history of smoking.
A 12-year-old girl with Beckwith-Wiedemann syndrome develops profound breast enlargement at the onset of puberty. Physical examination shows two distinct masses in the right breast and severe hyperplasia consuming the left breast. A photograph is shown. Which of the following is the most appropriate surgical management?
A) Hormone suppression therapy
B) Right lumpectomies; left mastectomy with skin reduction with application of nipple as skin graft
C) Right simple mastectomy; left mastectomy with skin reduction with application of nipple as skin graft
D) Right simple mastectomy; left simple mastectomy with sentinel lymph node biopsy
E) Right total mastectomy; left modified radical mastectomy
The correct response is Option B.
Epithelial hyperplasia is a benign pathological process. Therefore, modified radical mastectomy or sentinel lymph node biopsy would not be indicated in the absence of cancer. Mastectomy on the right is not indicated because the masses are discrete and separate from the normal breast parenchyma. As such, a viable breast mound can be preserved in this 12-year-old girl. Because of the expansive nature of her hyperplasia on the left, skin reduction is required to ensure smooth contour of her skin flaps. The nipple may be spared because of the benign disease. Reconstruction can be performed in a delayed fashion after development is complete. An adjustable expander/prosthesis can also provide a reasonable breast mound until definitive reconstruction is performed.
Hormone suppression therapy may temporarily slow the growth of the hyperplasia but could adversely affect this child’s normal development. It would not comprise definitive management.
Lactiferous ducts in the human embryo are derived from which of the following types of cell populations?
A) Endodermal
B) Epithelial
C) Mesenchymal
D) Neural crest
E) Pluripotential
The correct response is Option B.
As early as 6 weeks of gestation, discrete areas of epithelial proliferation occur on the ventral thorax. These areas evolve into buds that then canalize and form secondary buds that give rise to lactiferous ducts. These, along with secretory acini that originate in puberty, constitute the parenchyma of the breasts. These ducts are surrounded by mesodermal-derived mesenchymal cells, which in turn develop into the breast stroma composed of smooth muscle cells, capillary endothelial cells, and adipocytes. Neural crest cells give rise to pigmented cells and other structures but are not involved with embryogenesis of the breasts. Endodermal cell lines produce the lining of the gut and contribute to development of the liver and pancreas but not the breasts. Pluripotential cells do not comprise the parenchyma nor the stroma of the developing breast.
A 45-year-old woman who is obese is considering unilateral mastectomy and reconstruction of the left breast because of invasive ductal carcinoma. Which of the following patient characteristics is associated with the lowest risk for complications from a nipple-sparing mastectomy?
A) BMI of 41 kg/m2
B) Grade III ptosis of the breast
C) Nipple retraction
D) Tumor distance from nipple of 5 cm
E) Tumor size of 6 cm
The correct response is Option D.
Nipple-sparing mastectomy is increasing in popularity. To decrease the risk for surgical complications as well as oncologic complications, smaller tumors located further from the nipple in patients without morbid obesity or severe ptosis are considered better candidates for treatment with nipple-sparing mastectomy. Clinical involvement of the nipple, including retraction, would suggest that nipple-sparing mastectomy should not be performed.
References
A 37-year-old woman comes to the office for consultation regarding left breast reconstruction after mastectomy to treat breast cancer. Chemotherapy and radiation therapy are planned postoperatively. She wears a size 34D brassiere. Height is 5 ft 6 in (168 cm), and weight is 160 lb (73 kg). BMI is 25.8 kg/m2. She does not want abdominal scars. Autologous reconstruction with a transverse musculocutaneous gracilis (TMG) flap is planned. Which of the following is a disadvantage of using a TMG flap for this patient’s reconstruction?
A) Difficulty of flap harvest
B) High risk of donor site morbidity
C) High risk of fat necrosis
D) Small flap size
E) Variable vascular anatomy
The correct response is Option D.
For breast reconstruction, the TMG flap offers a valuable alternative therapy compared with other standard flaps from the lower abdomen, such as the transverse rectus abdominis musculocutaneous (TRAM) or DIEP flaps. The amount of tissue that can be transferred, however, is limited. The largest flap harvested in one large series weighed 420 g and measured 30 × 10 cm. The flap usually offers enough volume to reconstruct small- to mid-sized breasts. The major advantage of this flap as compared with other flaps, such as the gluteal flap or the perforator latissimus flap, is its constant vascular anatomy. Flap perfusion is always reliable, with low rates of fat necrosis and tissue similar in consistency with breast tissue. The donor scar is inconspicuous. The incision is comparable to incisions for a thigh lift and is well hidden. A distortion of the labia majora is typically not observed. However, as in thigh lifts, a lowering of the scar remains a possible problem. Functional donor-site morbidity after TMG flap harvest itself is low. After having clinically established the transverse approach for routine procedures, harvesting one flap is rapid and usually takes no longer than 30 minutes.
Which of the following hormones in the mammary gland is critical for breast development during puberty?
A) Cortisol
B) Insulin-like growth factor-1 (IGF-1)
C) Platelet-derived growth factor (PDGF)
D) Prolactin
E) Vascular endothelial growth factor-C (VEGF-C)
The correct response is Option B.
While pubertal breast development depends on an estrogen surge, it is important to remember that breast development occurs downstream. The surge triggers pituitary growth hormone to stimulate mammary gland production of insulin-like growth factor-1 (IGF-1). IGF-1, in turn, causes proliferation and anti-apoptosis through a signaling cascade. Disruption of any factor along this pathway will result in abnormal breast development. Laron syndrome, for example, is an autosomal recessive disorder in which growth hormone insensitivity can cause delayed puberty as well as short stature. While high levels of prolactin can cause breast growth (and are thought to be responsible for ultimate breast development in Laron syndrome), prolactin is not in the mammary glands themselves.
In addition to the critical role IGF-1 plays during breast development, it also plays an important role in breast development during pregnancy. High levels of IGF-1 are also associated with increased breast volume and early-onset breast cancer. High serum levels of IGF-1 are associated with both bad prognosis and increased mortality in breast cancer. Cortisol is a stress hormone not related to breast development. Platelet-derived growth factor has a primary role in blood vessel development, as does vascular endothelial growth factor (VEGF). VEGF-C has a function in lymphangiogenesis.
An otherwise healthy 41-year-old woman is scheduled to have bilateral prophylactic NSM for a BRCA1 mutation. Which of the following is an advantage of prepectoral implant reconstruction when compared with submuscular implant reconstruction?
A) There are significantly more acute postoperative surgical complications
B) There is a higher revision rate
C) There is a lower rate of animation deformity
D) There is a lower rate of upper pole rippling and implant palpability
E) There is more pain and functional impairment
The correct response is Option C.
The patient is a young, healthy, thin woman presenting for bilateral breast reconstruction following prophylactic mastectomies because of increased lifetime risk of breast cancer. She will maintain her entire breast skin envelope because she has planned nipple-sparing mastectomies and does not wish to increase her breast size with the operation. She has questions about the plane of implant placement above or below the pectoralis muscle and the answer choices provide the opportunity to counsel her on direct to implant versus staged expander to implant breast reconstruction.
Because of the lack of camouflage afforded by placement of the pectoralis muscle over the upper pole of the breast implant, prepectoral breast reconstruction is associated with increased rippling of the upper portion of the breast. This patient is thin according to BMI and so will have an increased risk of upper pole rippling that can be improved with subsequent fat grafting, provided she has adequate donor sites for fat harvest.
Several patient series reporting on early outcomes after prepectoral breast reconstruction have used a partially submuscular patient cohort as a comparison for acute postoperative complications. The overwhelming majority of these series showed no significant differences in acute postoperative outcomes and revision rates between techniques.
Postoperative functional assessments have demonstrated less pain and earlier return of function after prepectoral implant placement compared to submuscular implant placement. Re-siting of submuscular implants to a prepectoral plane have demonstrated resolution of animation deformity.
Algorithms for patient selection have slowly evolved over the past five years with the only constant recommendation being that the ideal candidate has a BMI less than 30 kg/m2. Earlier series emphasized mastectomy skin flap thickness, whereas more recent studies focus on skin flap viability. Earlier studies considered radiation treatment a contraindication, whereas more recent reports demonstrate the negative impact of radiation on pectoralis major fibrosis in submuscular reconstruction as compared with prepectoral implant placement.
A 54-year-old woman undergoes bilateral immediate tissue expander–based breast reconstruction. BMI is 36 kg/m2. On postoperative day 10, examination shows bilateral breast erythema; empiric antibiotics are started. In addition to Staphylococcusspecies, which of the following bacteria should be treated as the next most likely pathogen?
A) Bacteroides fragilis
B) Enterococcus faecalis
C) Mycobacterium marinum
D) Pasteurella multocida
E) Pseudomonas aeruginosa
The correct response is Option E.
After Staphylococcus aureus and S. epidermidis, Pseudomonas aeruginosa is among the the next most common sources of breast infections. It is common in hospitalized or immunocompromised patients, as well as patients with foreign body devices such as catheters or implants. It is a gram negative rod, and common antibiotic treatments include advanced β-lactams (piperacillin, ceftazidime), carbapenems, quinolones, and aminoglycosides. Dual coverage is often recommended in severe infections. In the case of breast implant infections, if there is not rapid improvement on antibiotic therapy, or if significant systemic symptoms develop (vital sign instability, high white blood cell count, fever, renal impairment), then surgical washout and device removal is mandatory. In patients without systemic symptoms, wash out and new implant placement can be an option in carefully selected and counseled patients. Other breast pathogens include Escherichia coli, Propionibacterium, and Corynebacterium.
More than 300,000 breast implant procedures are performed each year in the United States. In reconstructive cases, the infection rate averages 6% and the explantation rate 3% (range, 1.5 to 8%). Preventative measures include proper patient selection, preoperative MRSA management when carriers are suspected, routine presurgery chlorhexidine washes, proper antibiotic timing presurgery and continuation of antibiotics in implant reconstruction cases for at least 24 hours (though the optimal treatment duration has not yet been determined).
None of the other bacteria listed are common in breast infections, though all are common pathogens. Bacteroides are anaerobic gram-negative rods that are common in gut flora and feces.
Enterococcus faecalis is a frequent cause of nosocomial infection, with a high prevalence of multi-drug resistance. It is a gram-positive coccus, and is not commonly seen in breast surgery patients as it primarily colonizes the digestive tract.
Mycobaterium marinum is a rare pathogenic cause of hand infections from injuries that occur in aquatic environments.
Pasteurella multocida is a frequent cause of animal bite infections, particularly from cats and dogs.
A 52-year-old woman is evaluated for breast reconstruction after modified radical mastectomy with adjuvant chemotherapy and radiation therapy 18 months ago. Her last radiation treatment was 8 months ago. BMI is 29 kg/m2. Examination today shows hyperpigmentation of the right chest wall with no redundancy of the mastectomy skin flaps. Her contralateral breast is a D cup with grade III ptosis. Which of the following methods will create the best symmetry for this patient?
A) Deep inferior epigastric perforator flap
B) Gel breast implant and acellular dermal matrix
C) Latissimus dorsi musculocutaneous flap
D) Tissue expander and acellular dermal matrix placement with planned staged exchange for permanent gel implant
E) Tissue expander placement alone with planned staged exchange for permanent gel implant
The correct response is Option A.
The deep inferior epigastric perforator flap would give the patient autologous tissue reconstruction with ample tissue for skin resurfacing and soft tissue for volume. In this radiated patient with a tight skin envelope, a tissue expander/implant, with or without acellular dermal matrix, would be difficult to create an appropriately ample skin envelope and the patient would be at higher risk for wound-healing problems and capsular contracture. The latissimus dorsi musculocutaneous flap, although an autologous tissue reconstruction, would have insufficient volume to adequately match this patient’s contralateral side. It would have to be combined with an implant.
For reconstruction mammaplasty, which of the following is an advantage of the extended latissimus dorsi flap over the standard latissimus dorsi myocutaneous flap?
(A) Better flap perfusion
(B) Decreased need for breast implant
(C) Fewer donor-site seroma
(D) Less sacrifice of latissimus dorsi muscle
(E) Smaller donor-site scar
The correct response is Option B.
The latissimus dorsi (LD) myocutaneous flap was one of the first methods of breast reconstruction ever described. However, with the increasing popularity of transverse rectus abdominis myocutaneous (TRAM) flap breast reconstruction, for many, the LD flap has become a secondary choice for autologous breast reconstruction. One reason for this has been that the standard LD flap alone often does not provide sufficient volume for breast reconstruction. In fact, it is a common practice to combine the LD flap with a prosthesis to achieve adequate breast volume.
Variations of the LD flap have been described to increase its volume and avoid the addition of a prosthesis. The first “extended” LD (ELD) flap was described by Hokin in 1983 and included lumbar fat extensions of the LD flap. Others have used the buried de-epithelialized LD myocutaneous flap for breast reconstruction. McCraw and Papp modified this technique by using a fleur-de-lis skin paddle design to carry additional fat on the surface of the LD muscle, creating the totally autogenous LD breast reconstruction.
The design of the ELD flap has evolved to include the parascapular and scapular “fat fascia” in addition to the lumbar fat for additional volume. The main advantage of the ELD flap is that it can provide autogenous tissue to replace breast volume without an implant and with acceptable donor site contour and scar. Because the ELD flap transports additional tissue from the back so a breast prosthesis is not required, this avoids all potential problems associated with implants, e.g., development of capsular contracture and deformation of the reconstructed breast.
With ELD flaps, donor site complications, including seroma formation and wound necrosis, can be a significant problem.
A 14-year-old girl has absence of the nipple and lack of development of the right breast. The left breast has normal shape and normal nipple-areola complex and fits a B-cup brassiere. Family history includes normal breast development in the parents and siblings. On physical examination, both pectoralis muscles are present and fully developed. No abnormalities of the hands are noted. Which of the following is the most likely diagnosis?
(A) Anterior thoracic hypoplasia
(B) Congenital absence of the breast
(C) Hypoplasia of the breast
(D) Poland syndrome
(E) Tubular breast deformity
The correct response is Option B.
Congenital absence of the breast is defined by the absence of the nipple and mammary gland. This rare genetic condition is highly heterogeneous in presentation and inheritance.
Anterior thoracic hypoplasia is defined by unilateral sinking of the anterior chest wall, hypoplasia of the breast, superior location of the nipple-areola complex, and normal pectoralis muscles. In hypoplasia of the breast, both nipples are present. In Poland syndrome, the defining feature is partial or complete aplasia of the sternocostal head of the pectoralis muscle and deformity of the breast and upper extremity. Tubular breast deformity involves constriction at the base of the breast, hypoplasia of the breast, and herniation of tissue into the nipple-areola complex.
A 42-year-old woman with a history of a cesarean delivery from a low-transverse abdominal incision is scheduled to undergo a unilateral deep inferior epigastric perforator (DIEP) flap breast reconstruction. BMI is 28 kg/m² and the distance from nipple to sternal notch is 24 cm per side. This patient’s history of cesarean delivery is most likely to have which of the following effects?
A) Decreased abdominal seroma
B) Decreased flap venous congestion
C) Increased flap arterial thrombosis
D) Increased flap fat necrosis
E) No overall effect
The correct response is Option B.
Pfannenstiel incisions are the preferred access for cesarean deliveries. They are not a contraindication for abdominal-based flaps for breast reconstruction because the deep inferior epigastric circulation is not disturbed. However, the superficial epigastric circulation may be divided. The net result appears to be a more robust venous circulation with a protective effect against fat necrosis in the flap. This venous division causes a delay-type phenomenon—during healing increased branches are formed from the superficial epigastric circulation, and there is some evidence of new connections to the venae comitantes of the deep epigastric venous circulation.
There is evidence, however, for an increased rate of abdominal healing problems, including seroma (15% versus 6%), wound healing problems, and fat necrosis in the abdomen. There is no evidence for an effect on the arterial circulation of the flap.
A 54-year-old woman with BRCA mutation is scheduled for a bilateral nipple-sparing mastectomy. Which of the following would lead to the highest risk for postoperative nipple necrosis?
A) Grade 2 breast ptosis
B) Periareolar mastectomy incision
C) Previous excisional breast scar
D) Tumor greater than 1.5 cm
E) Use of smooth round gel implants
The correct response is Option B.
Nipple-sparing mastectomy and direct to implant reconstruction is becoming increasingly popular in the setting of prophylactic mastectomies. The criteria for nipple-sparing mastectomies have been increased to tumors not involving the nipple-areola complex, with some surgeons requiring a distance of 2 cm from the nipple and others espousing just a clean surgical margin at the nipple. Nipple-sparing mastectomies do not require recreating the breast envelope and therefore direct to implant reconstruction is possible. Increased risk for incidence of nipple necrosis in the setting of direct to implant reconstruction is associated with mastectomy incision involving the areola. The use of smooth round gel implants, grade 2 ptosis, tumor size, or previous breast biopsy scars are not associated with nipple necrosis.
A 46-year-old woman who is 5 ft 7 in (170 cm) tall and weighs 135 lbs (61 kg) is evaluated one year following bilateral nipple-sparing mastectomy and immediate reconstruction with placement of 350-mL smooth, round silicone gel implants beneath the pectoralis major muscle. Since the surgery, she has experienced hyperdynamic deformity of her breasts. On physical examination, the breast reconstruction appears natural, and there is significant movement of the breasts when the patient flexes her chest. Which of the following is the most appropriate management for this patient?
A) Inject botulinum toxin into the pectoralis major muscle
B) Inject triamcinolone-40 into the areas of tenderness using ultrasound guidance
C) Move the implants to the prepectoral plane and cover them fully with acellular dermal matrix
D) Perform a breast MRI to assess for rupture of the implants
E) Refer the patient to a physical therapist for range of motion, massage, and ultrasound treatments
The correct response is Option C.
This patient is experiencing significant movement because her implants were placed beneath the pectoralis major muscles. While reconstruction options are limited in this otherwise healthy and very thin patient who is not a good candidate for fat grafting or pedicled or free tissue transfer, placing implants over the pectoralis major muscles and covering the implants fully with acellular dermal matrix would be the most appropriate method of reconstructing her breasts and addressing her concerns.
Physical therapy and muscle relaxants are unlikely to produce long-term improvement. An MRI would likely be nondiagnostic, and even if her implants were ruptured, change to a prepectoral plane is still indicated. Botulinum toxin type A is likely not as effective for long-term significant improvement as reoperation. Triamcinolone would not be effective for hyperdynamic deformity.
A 45-year-old woman comes to the office for consultation regarding mastectomy and immediate breast reconstruction because of recurrent right-sided breast cancer. She underwent breast-conserving therapy and radiation therapy 5 years ago. BMI is 23 kg/m2. The patient is otherwise healthy, and she does not smoke cigarettes. On examination, the breast is soft with obvious fibrosis. Use of which of the following has the highest risk for reconstructive failure in this patient?
A) Deep inferior epigastric artery perforator (DIEP) flap
B) Free transverse rectus abdominis musculocutaneous (TRAM) flap
C) Latissimus flap with prosthesis
D) Pedicled TRAM flap
E) Tissue expander and prosthesis
The correct response is Option E.
Pre-reconstruction radiation is typically seen in two groups of patients: those who underwent mastectomy without reconstruction followed by radiation therapy, or those who underwent breast-conserving therapy and radiation with recurrence or new cancer. Although these patients may present with a reasonable skin envelope, complication rates associated with tissue expander/prosthesis have been reported as high as 70%, with a 40% rate of failure or conversion to flap. Additionally, aesthetic outcomes in most patients who completed expander/prosthesis reconstruction after previous radiation therapy were deemed good or very good compared with the majority of non-irradiated patients who achieved excellent results. Tissue expander/prosthesis can be considered as an option for patients with a history of previous irradiation who wish to avoid the scars and recovery of flap-type reconstructions; however, they should be counseled of the high risks of complications.
According to the 2017 US breast cancer mortality data, three states with appropriately powered data achieved statistically equivalent mortality rates between non-Hispanic African American and non-Hispanic Caucasian women. Which of the following factors was most likely implicated in the improvement in mortality rates in the non-Hispanic African American women in these states?
A) Non-Hispanic African American women are more affluent
B) Non-Hispanic African American women are younger
C) Non-Hispanic African American women have fewer “triple-negative” aggressive breast cancers
D) Non-Hispanic African American women have more access to screening mammography and in situ diagnosis
The correct response is Option D.
Social determinants of health include where a person resides, what exposures are present, what diet is eaten, how much stress is present, and other lifetime experiences. Social determinants of health affect who gets disease and how well they can be cared for when they have disease.
According to the 2020 cancer statistics, death from female breast cancer has declined overall by 40% in the United States since 1989.
The 2017 breast cancer statistics look at breast cancer in a state-by-state fashion demonstrating that those gains in survival have not been uniformly spread among the populations, with non-Hispanic African American women suffering higher death rates by age group, staging, and oncotyping groups.
While there are no data to discuss why non-Hispanic African American women tend to develop cancer younger or develop more “triple negative” breast cancers, there has been evidence presented in the 2017 breast cancer statistics that better access to care may improve the stage at diagnosis. In seven states, the mortality rates were statistically the same. In three of those states—Massachusetts (an early state to establish mandatory health insurance), Connecticut, and Delaware—there were sufficient cohort numbers of non-Hispanic African American women with an in situ diagnosis, which is used as a proxy for access to screening mammography. In the four other states, there was statistical equivalence, but they were underpowered. The implication of these data is that better access to mammography/care lowers mortality for non-Hispanic African American women with breast cancer. These studies did not address affluence, but they did rule out age as a factor.
Improvement in which of the following is an advantage of nipple-areola complex reconstruction?
A) Breast feeding ability
B) Breast mound shape
C) Reactivity to touch
D) Satisfaction with reconstruction
E) Sexual sensation
The correct response is Option D.
Satisfaction with reconstruction, quality of life, and feeling of completeness with reconstruction have all been shown to be linked to nipple and/or areola reconstruction. The reconstruction does not restore nipple function; therefore, breast feeding, sexual sensation, and reactivity to touch are not accomplished with nipple and/or areola reconstruction. Often the nipple reconstruction, if using local tissue techniques, may actually flatten the anterior aspect of the breast and is not thought to positively impact the breast mound shape.
Which of the following is the primary advantage of performing breast reconstruction with an extended latissimus dorsi myocutaneous flap instead of the standard latissimus dorsi myocutaneous flap?
(A) Decreased donor site scarring
(B) Decreased incidence of seroma at the donor site
(C) Decreased need for adjuvant breast implantation
(D) Decreased need for division of the humeral tendon attachment
(E) Decreased risk for flap loss
The correct response is Option C.
The primary advantage of the extended latissimus dorsi flap is the autogenous tissue it provides, which can be used to enhance breast volume without the need for an implant. The standard latissimus dorsi myocutaneous flap was one of the first created for breast reconstruction. However, with the emergence of the TRAM flap, the latissimus dorsi flap has become a secondary choice for autologous reconstruction for many surgeons because it often does not provide sufficient volume and is, instead, often used in combination with an implant.
Variations of the latissimus dorsi flap have been created to increase volume and obviate the need for an implant. The first extended latissimus dorsi flap, described in 1983, was designed to include fat and fascia from the parascapular and scapular regions and fat from the lumbar spine. This transfer of additional tissue from the back eliminates the need for an implant; therefore, potential complications associated with implants, such as the development of capsular contracture and deformation of the reconstructed breast, are avoided. The technique was later modified by creating a fleur-de-lis skin paddle, which carries additional fat on the surface of the latissimus dorsi muscle and is thus a totally autogenous latissimus dorsi flap.
The contour and scarring of the donor site following flap transfer are acceptable. However, other complications, such as seroma formation and wound necrosis, can be a significant concern. Division of the humeral tendon allows for
increased mobility of the flap. Partial flap and/or fat necrosis are more commonly associated with the extended flap than the standard flap.
A 62-year-old woman undergoes breast reconstruction using autologous tissue from the abdomen. Intraoperatively, use of a perforator flap is found to be impossible because of multiple small nondominant perforators. Conversion to a delayed pedicled transverse rectus abdominis muscle flap is planned. Ligating which of the following vessels in this stage will best facilitate future viability of the tissue transferred in the next stage?
A) Deep inferior epigastric
B) Hypogastric
C) Internal mammary
D) Superficial inferior epigastric
E) Superior epigastric
The correct response is Option A.
Pedicled transverse rectus abdominis muscle (TRAM) flaps are based on the superior epigastric system, which is often less robust than the deep inferior epigastric system. Therefore, surgical delay by ligation of the deep inferior epigastric system may facilitate overall viability of the transferred tissue. Ligation of the superior epigastric system would make a pedicled TRAM flap unlikely to survive. The internal mammary ligation may also interrupt blood supply to the superior epigastric system, and even if the tissue is fed through collaterals, it would not strengthen the flap. Division of the superficial inferior epigastric system might also help, but it is not as critical as ligation of the deep inferior epigastric system. The hypogastric system does not have a direct impact on the pedicled TRAM tissues.
A 43-year-old woman is scheduled for reconstruction of the right breast with a latissimus dorsi flap. She is at greatest risk for which of the following complications?
(A) Chronic chest wall pain
(B) Dorsal wound dehiscence
(C) Partial flap necrosis
(D) Seroma
(E) Stiffness in the ipsilateral shoulder
The correct response is Option D.
The most common complication in the scenario described is seroma formation, which is estimated to be as high as 35% to 60%. Other less common complications include chest wall pain (6%), dorsal wound dehiscence (4%), partial flap necrosis (1% €“7%), and shoulder stiffness (1%).
A 45-year-old woman presents with right breast cancer and is planning a nipple-sparing mastectomy and tissue expander placement. She is specifically interested in a carbon dioxide–based expander. Which of the following is a disadvantage of this device compared with a saline tissue expander?
A) Extrusion
B) Inability to deflate
C) Increase in wound dehiscence
D) Increase in wound infection
E) Possible device dislocation
The correct response is Option B.
The carbon dioxide-based tissue expander (AeroForm) is a fixed-volume device and has an inability to deflate the expander.
In a prospective, multicenter, randomized controlled trial comparing carbon dioxide–based expanders and saline tissue expanders, there were no statistically significant differences in rates of wound infection, extrusion, device dislocation, or wound dehiscence. Advantages of the carbon dioxide–based expander include a more rapid expansion process and a shorter time to implant exchange. The device is self-contained and patient-controlled, so there are no needles required and possibly fewer physician office visits.
Six hours after undergoing breast reconstruction with a free TRAM flap, a 35-year-old woman has swelling of the reconstructed area and bluish discoloration of the skin. A photograph is shown above. Capillary refill time is increased. Findings on Doppler ultrasonography of the flap are within normal limits.
Which of the following is the most appropriate next step in management?
(A) Fluid resuscitation
(B) Anticoagulant therapy
(C) Release of sutures and observation
(D) Operative exploration
The correct response is Option D.
This 35-year-old woman who underwent breast reconstruction now exhibits signs of venous outflow obstruction, including increased capillary refill time, dark or bluish discoloration of the skin, and swelling of the affected area. In addition, complete venous obstruction does not always manifest as abnormal findings on Doppler ultrasonography, and the Doppler signal may indeed fall within normal limits in many affected patients. Therefore, in the presence of these clinical signs, the only appropriate option is to perform repeat operative exploration.
Fluid resuscitation and anticoagulant therapy are not appropriate treatment of venous obstruction in patients who have undergone free tissue transfer. In the same way, release of the sutures will not resolve the obstruction.
An otherwise healthy 16-year-old girl comes to the office because of a painless mass in the left breast. Physical examination of the left breast discloses a circumscribed firm, rubbery, 3-cm mass without overlying skin changes, and no axillary lymphadenopathy. Results of a pregnancy test are negative. Which of the following is the most likely diagnosis?
A) Common fibroadenoma
B) Giant fibroadenoma
C) Lactating adenoma
D) Phyllodes tumor
E) Tubular adenoma
The correct response is Option A.
A common fibroadenoma is the most likely diagnosis of this patient. Common fibroadenoma is the most common breast tumor in adolescent females and present between the ages of 14 and 16.
Juvenile fibroadenoma is a variant of fibroadenoma and is usually seen in adolescents and young adults. It is associated with a normal stromal/epithelial balance, which distinguishes it from phyllodes tumor, and has both stromal and epithelial hyperplasia. In addition, juvenile fibroadenomas are characterized by rapid growth.
A giant fibroadenoma is a clinical diagnosis, rather than a pathologic diagnosis. It is characterized by its size, usually greater than 5 cm.
Complex fibroadenoma is characterized by fibrocystic changes on glandular tissue with underlying features of common fibroadenoma on pathologic analysis.
A tubular adenoma has glandular proliferation on pathologic analysis, and while it is a subtype of fibroadenoma, it is not as common as common fibroadenoma. A lactating adenoma, similarly, is defined by the presence of secretory hyperplasia of lobules on pathologic analysis. Lactating adenomas are so defined because of the histologic presence of secretory hyperplasia, and they commonly occur in pregnant or lactating women. Many lactating adenomas will spontaneously regress.
A phyllodes tumor is typically a large, rapidly growing lesion and can be either benign or malignant. It is rare in adolescents, but when found, is usually aggressive. Phyllodes tumor is related to fibroadenoma and is distinct from other forms of breast cancer. Treatment is wide local excision.
A 48-year-old woman undergoes radical resection of the left breast, left hemisternectomy, four-rib resection, and visceral pleurectomy because of invasive inflammatory breast cancer extending through the thoracic wall into the mediastinum and chest. Examination of specimens obtained from intraoperative frozen section biopsies shows no residual disease. Postoperative adjuvant chemotherapy and radiation therapy to the area are planned. An intraoperative photograph of the 17 × 17-cm defect is shown. Which of the following is the most appropriate technique for soft-tissue reconstruction?
A) Left latissimus muscle flap with skin graft
B) Omental flap with skin graft
C) Left rectus turnover flap with skin graft
D) Reverse abdominoplasty flap
E) Right TRAM flap
The correct response is Option E.
The most appropriate option for this patient would be a right TRAM flap. The defect encompasses a composite defect of the left side of the sternum, including the left internal mammary artery, four contiguous ribs, and the entire left breast. The question specifically asks for soft-tissue thoracic reconstruction. The analysis of the defect yields a very large soft-tissue requirement. Given this, the most logical choice of flaps to provide this amount of soft tissue on a reliable pedicle would be a right (contralateral) TRAM flap. Furthermore, this flap could also be shaped to provide a breast mound versus all the other choices.
A left latissimus muscle flap with skin graft is not the optimal choice as it would not provide sufficient soft-tissue coverage with its associated skin paddle without undue donor site morbidity in this particular patient with a large defect that spans parasternal and lateral chest wall. As a muscle-only flap with skin graft, however, it may be considered as a backup option in case of primary flap failure.
An omental flap with skin graft is an option for reconstruction of this defect; however, it would not be the most appropriate option in this case as it would necessitate an otherwise unnecessary intra-abdominal procedure and would not reshape a breast mound. It has been shown, however, to be optimal in contaminated areas due to its associated lymphatic vessels and nodes associated with its pedicle, so in different circumstances, this may be a preferred choice.
A left rectus turnover flap with skin graft would not be a good option for two reasons: the left internal mammary artery has been harvested, thereby compromising the superior epigastric vessel on which this flap would be based; and it would not cover the entirety of the defect.
A reverse abdominoplasty flap would not be able to reach the full superior extent of the defect.
A 21-year-old woman comes to the office for consultation regarding a palpable lump in the left breast that she first noticed 4 months ago. Phyllodes tumor of the breast is diagnosed. Which of the following is the most appropriate management at this time?
A ) Observation and incisional biopsy
B ) Neoadjuvant chemotherapy
C ) Local excision and annual surveillance
D ) Wide local excision and radiation therapy
E ) Mastectomy with immediate reconstruction
The correct response is Option C.
Phyllodes tumors of the breast were once called cystosarcoma phyllodes because of the fleshy nature of the tumor. Because most tumors are benign, the name may be misleading. Thus, the favored terminology is now phyllodes tumor.
Phyllodes tumor is the most commonly occurring nonepithelial neoplasm of the breast but represents only about 1% of tumors in the breast. It has a sharply demarcated, smooth texture and is typically freely movable. It is a relatively large tumor; the average size is 5 cm. However, lesions greater than 30 cm have been reported. Approximately 90% of phyllodes tumors are benign and approximately 10% are malignant.
In considering excision, the tumor-to-breast ratio should be small enough to allow for segmental excision of the tumor and possible reduction mammaplasty technique (eg, inferior pedicle for a superior tumor or a superior pedicle for an inferior lesion). Annual surveillance and follow-up is recommended because the pathologic appearance does not always predict the clinical behavior.
A 50-year-old woman comes to the office 6 weeks after undergoing right mastectomy and immediate placement of a tissue expander. She reports swelling and redness of the right breast. A photograph is shown. Which of the following factors is most predictive of implant salvage failure in this patient?
A) Culture positive for Pseudomonas species
B) Elevated body mass index
C) Periprosthetic seroma
D) Presence of cellulitis
E) Previous irradiation
The correct response is Option A.
Immediate implant-based reconstruction has become increasingly popular over the past two decades, accounting for over 70% of all reconstructions in the United States. The benefits of immediate reconstruction are numerous, including decreased recovery/number of required procedures and increased patient psychological well-being and aesthetic outcome. However, the complication (seroma, mastectomy flap necrosis, loss of implant, and infection) rates after implant-based reconstruction remain relatively high. Infection rates in the reported literature range from 2.5 to 24%.
Historically, periprosthetic infection or implant exposure mandated immediate implant removal. However, numerous studies over the past several decades have demonstrated implant salvage rates of 37.3 to 73% depending on the methods employed. Several studies have looked at the predictive factors that increase the risk of a failed salvage attempt. Salvage was typically defined as administration of systemic antibiotics (oral or intravenous), removal of the infected implant, partial/total capsulectomy, pocket curettage, implant pocket irrigation with antibiotic solution, and placement of a new device.
Factors associated with implant salvage failure include an elevated white blood cell count, elevated temperature, deep-seated pocket infection (purulent periprosthetic fluid), and atypical pathogens such as methicillin-resistant Staphylococcus aureus (MRSA) and Pseudomonas species. Spear et al. showed that 93.9% of mild implant infections (localized cellulitis) could be salvaged compared with a 30% salvage rate in the severe infection group. Factors such as smoking, chemotherapy, previous irradiation, mastectomy skin necrosis, increased BMI, and use of acellular dermal matrix (ADM) have demonstrated increased rates of implant-related infections, but these factors have not been demonstrated to increase the risk of implant salvage failure.
A healthy 45-year-old woman with a history of breast malignancy underwent bilateral mastectomy and reconstruction with tissue expanders followed by exchange for cohesive silicone gel implants eight years ago with routine postoperative MRI surveillance. She comes to the office to report pain and tightness in the right breast that has gradually increased over the past month. On examination, temperature is 36.8°C (98.2°F), blood pressure is 112/76 mmHg, and heart rate is 68 bpm. The right breast appears fuller than the left breast; otherwise, the right implant is in a symmetric position with the left side. The skin is otherwise normal in appearance, and there is no tenderness on palpation. Which of the following is the most appropriate next step in management?
A) MRI of the right breast to assess the integrity of the implant
B) One week of an oral antibiotic and prednisone taper
C) Operative exploration, culture, and replacement of implant
D) Referral of the patient back to her medical and surgical oncologists
E) Ultrasound of the right breast and fine-needle aspiration of any fluid
The correct response is Option E.
Patients that present with a late seroma should be evaluated for possible Breast implant associated Anaplastic Large Cell Lymphoma (BI-ALCL). A late seroma is usually accepted as occurring 1 year following surgery: however there are cases of BI-ALCL seroma that have presented as early as 4 months.
The first step in evaluation for BI-ALCL is an ultrasound followed by fine needle aspiration is indicated. The fluid requires evaluation beyond routine cell cytology. Immunohistochemistry test for CD30 was the most commonly positive marker for BI-ALCL. Immunohistochemistry stains specific antigens in cells by binding to this antigen in an antibody/antigen reaction. The specific stain can then be seen under light microscopy. CD30 antibody labels anaplastic large cell lymphoma cells. CD30 is a transmembrane cytokine receptor belonging to the tumor necrosis factor receptor family and characteristically stains ALCL cells.
MRI for implant integrity and referral to her Oncologist may be needed but it is not the most appropriate next step. BIA-ALCL needs to be ruled out. Immediate operative exploration is not indicated before fluid aspiration and immunohistochemistry evaluation. Antibiotics and prednisone is not indicated in this patient without evidence of infection or inflammation (red breast syndrome).
A 41-year-old woman comes to the office because of an invasive ductal carcinoma of the left breast. On mammography, the tumor is 3 cm from the nipple and measures 4 cm. A left-sided lateral periareolar scar extending from the 12 o’clock to the 3 o’clock position from a previous biopsy is noted. The patient wishes to undergo a nipple-sparing mastectomy. Which of the following findings places the patient at greatest oncologic risk, including risk for de novo or recurrent cancer or inadequate surgical margins, with this procedure?
A) Distance of tumor to nipple
B) Patient age
C) Presence of the periareolar scar
D) Size of tumor
E) Type of tumor
The correct response is Option D.
As surgical approaches to breast cancer treatment have evolved, nipple-sparing mastectomy (NSM) has emerged as an alternative to other approaches. It was initially used for prophylactic mastectomies, and patients reported increased satisfaction and body image with nipple-areola complex (NAC) preservation. The role of NSM has been expanded to therapeutic mastectomy, and with that there has been increased research in the oncologic safety of this approach.
Studies have evaluated therapeutic NSM in the context of invasive ductal carcinoma, invasive lobular carcinoma, and ductal carcinoma in situ. The type of cancer does not appear to be associated with the oncologic safety of NSM. Several studies have demonstrated an inverse association between NAC involvement and distance of the tumor from the nipple. While these studies have varied in their distance cutoffs, nipple involvement is reported to be over 50% when the tumor-nipple distance is less than 2 cm, as noted in one study. There is a direct correlation between tumor size and NAC involvement—the same study cited data that when the tumor was greater than 4 cm, the likelihood of nipple involvement was greater than 50%.
One published screening algorithm for plastic surgery includes tumor size less than 3 cm, and tumor location greater than 2 cm from the nipple as criteria for NSM candidacy.
A periareolar scar, if large, may compromise the blood supply to the NAC. Acceptable incisions for NSM, however, include a periareolar incision of 25 to 50%.
In women undergoing prosthetic breast reconstruction complicated by an expander/implant infection, which of the following is the most common gram-negative bacteria isolated from cultures?
A) Escherichia coli
B) Klebsiella
C) Proteus
D) Pseudomonas
E) Serratia
The correct response is Option D.
Tissue expander/implant-based breast reconstruction remains the most common form of reconstruction after mastectomy. One of the most potentially devastating complications of this form of breast reconstruction is an implant infection with need for removal of the expander/implant. The mean reported incidence of implant infection after breast reconstruction is 8%, with a range of 1 to 35%. When cultures are obtained, the most common causative bacteria on microbiology examination are gram-positive organisms (41 to 83%), specifically, Staphylococcus species (56 to 76.5%). Gram-negative bacteria accounted for 15.3 to 28.6%, with Pseudomonas (10.7 to 14%) being the most common gram-negative bacteria present on microbiology examination.
A 40-year-old woman is scheduled to undergo reconstruction of the right breast via a free TRAM flap. She has smoked two packs of cigarettes daily for the past eight years. This patient=s smoking history increases her risk of which of the following postoperative complications?
(A) Hematoma
(B) Mastectomy flap necrosis
(C) Seroma
(D) TRAM flap loss
(E) Vessel thrombosis
The correct response is Option B.
One study found no significant increased risk of flap loss, vessel thrombosis, or fat necrosis in free TRAM flaps performed on smokers. However, smokers did have an increased risk over their nonsmoking counterparts in the area of mastectomy skin flap necrosis, abdominal hernia, and abdominal flap necrosis.
Another study of 569 free TRAM flaps found that smokers had an increased rate of fat necrosis, wound infection, abdominal flap necrosis, and mastectomy flap necrosis. There was no correlation with free flap loss, thrombosis, hematoma, or seroma.
By using the deep inferior epigastric vessels, which are the primary blood supply to the TRAM flap, risks to the flap itself are minimized even in smokers. But smoking continues to increase the risk of donor site morbidity and native breast skin necrosis.
A 42-year-old woman is evaluated because of an invasive cancer of the right breast. Which of the following best approximates the likelihood that this patient’s cancer is associated with the BRCA1 or BRCA2 genes?
A) 1%
B) 10%
C) 25%
D) 40%
E) 75%
The correct response is Option B.
Among familial breast cancers, 5 to 10% are considered to be hereditary. These familial breast cancers are linked to specific mutations on a cancer susceptibility gene. The breast cancer susceptibility genes (BRCA) belong to a class of genes known as tumor suppressors. In normal cells, BRCA1 and BRCA2 genes stabilize the DNA and prevent uncontrolled cell growth.
A woman’s lifetime risk of developing breast and/or ovarian cancer is greatly increased if she inherits a mutation on BRCA1 or BRCA2 genes. BRCA1- and BRCA2-related breast cancers occur in younger women and are often associated with estrogen receptor-negative tumors.
A 46-year-old woman with ductal carcinoma in situ is scheduled to undergo right mastectomy. Immediate reconstruction with a tissue expander and acellular dermal matrix (ADM) is planned. Which of the following is an expected outcome with use of ADM?
A) Decreased formation of seroma
B) Decreased incidence of hematoma
C) Decreased risk of infection
D) Increased capsule contracture
E) Increased initial fill of the expander
The correct response is Option E.
According to Sbitany, et al, acellular dermal matrix (ADM) allows for a greater initial fill of saline. This potentially improves cosmetic outcome, as it better capitalizes on preserved mastectomy skin for reconstruction. Sbitany, et al, concluded that ADM-assisted prosthesis breast reconstruction has a safety profile no worse than that of complete submuscular coverage but offers the benefit of fewer expansions and the potential for more predictable secondary revisions.
ADM has enhanced prosthesis-based reconstruction and remains useful in immediate prosthetic breast reconstruction. However, it has been found to have higher rates of postoperative seroma and infection. It has also been reported to decrease capsule contracture.
A 45-year-old woman with T3 N0 invasive ductal carcinoma in the inferior pole of the left breast is scheduled to undergo segmental mastectomy and subsequent radiation therapy. She currently wears a size 36DDD brassiere and is willing to accept any cup size from C to DDD. Which of the following interventions will yield the best cosmetic result in this patient?
(A) Bilateral reduction mammaplasty
(B) Implantation of a prosthesis in the left breast and mastopexy of the right breast
(C) Latissimus dorsi musculocutaneous flap reconstruction of the left breast and mastopexy of the right breast
(D) TRAM reconstruction of the defect
(E) No reconstruction
The correct response is Option A.
In a patient with large breasts, in whom a partial mastectomy is required, reduction mammaplasty is a very good option. This procedure will potentially relieve symptoms of macromastia, reduce the amount of breast tissue present in both breasts, and offer the best aesthetic outcome.
Implantation of a prosthesis in a breast that will undergo radiation therapy increases the risk for complications and would likely lead to a less symmetrical result.
The latissimus dorsi (or TRAM) flap could be used for immediate partial reconstruction but is ideal for delayed reconstruction of partial mastectomy defects. Reduction mammaplasty does not preclude future reconstruction options, but a latissimus flap reconstruction would.
With a T3 tumor, a significant portion of the lower pole of the breast is removed to obtain clear margins. Even though the patient described has moderate to large breasts, there is a high likelihood that she will develop a deformity of the left breast and asymmetry with the right breast if no reconstruction is performed.
A 13-year-old girl is brought to the office by her parents because her left breast is not developing. On examination, both nipples and areolae are present. The breasts are asymmetric; the left breast is considerably smaller. The left anterior axillary fold is absent. This patient is most likely to have which of the following additional developmental differences?
(A) Ambiguous genitalia
(B) Craniosynostosis
(C) Microtia
(D) Pectus excavatum
(E) Syndactyly
The correct response is Option E.
This patient has Poland syndrome, which is characterized by unilateral breast or nipple hypoplasia, unilateral absence of the sternal head of the pectoralis major muscle, absence of the pectoralis minor muscle, and ipsilateral syndactyly or hypoplasia of the ipsilateral extremity. In severe Poland syndrome, rib anomalies also occur.
Ambiguous genitalia, craniosynostosis, microtia, and pectus excavatum are not associated with Poland syndrome.
A 30-year-old woman comes to the office because of pain in the left breast. Two weeks ago, she underwent core needle biopsy of a breast mass that was diagnosed as benign. Family history does not include breast cancer. She does not smoke cigarettes. On examination, the left breast is erythematous and tender to palpation, and the skin of the breast is retracted laterally. There is a palpable, rope-like mass that courses longitudinally along the breast. Which of the following is the most appropriate diagnosis and treatment of this lesion?
A) Breast abscess, perform incision and drainage of the mass
B) Fibrocystic changes, perform biopsy
C) Mastodynia, treat with 10-day course of broad-spectrum antibiotics
D) Nipple papilloma, perform diagnostic mammography
E) Superficial thrombophlebitis, manage with analgesics
The correct response is Option E.
Mondor disease, or superficial thrombophlebitis of the breast, involves the superficial veins of the breast and anterior chest wall. It may occur following surgery, core biopsy, irradiation, or trauma. Clinical manifestations include pain, redness and swelling, and the presence of a thickened tender cord. This condition usually resolves in 4 to 6 weeks with symptomatic treatment using pain relief.
Nipple papillomas may be identified as a mass on breast imaging or may be found incidentally. They frequently present with bloody nipple discharge. While not concerning in and of themselves, these lesions may harbor areas of atypia or ductal carcinoma in situ, and are treated with core needle biopsy.
While a breast abscess is possible after a diagnostic procedure such as a biopsy, it would present as localized swelling, tenderness, and induration. The skin retraction and rope-like mass would not be present.
Fibrocystic changes in the breast present as a solitary mass and may cause patients to seek medical attention because of associated pain. They may fluctuate in size and tenderness during a patient’s menstrual cycle. Because no breast mass can be definitively declared benign on physical examination alone, imaging and/or biopsy may be considered.
Breast pain in the absence of a finding on physical examination may have a number of causes including menstrual changes, breast hypertrophy, diet, hormone replacement therapy, ductal ectasia, mastitis, malignancy, and hidradenitis. While the patient does have breast pain, the other physical findings rule out mastodynia alone.
A 48-year-old woman comes to the office for consultation regarding reconstruction of the right breast after mastectomy because of cancer. The patient is concerned about maximizing the aesthetic result and minimizing any donor site deformity. Physical examination shows a well-healed chest wall and a B-cup left breast with Grade 3 ptosis. Soft-tissue reconstruction with a superior gluteal artery perforator (SGAP) free flap is planned. Which of the following is a disadvantage of this procedure?
(A) Difficulty molding the gluteal fat
(B) Gait dysfunction
(C) Inability to fully hide the donor scar
(D) Inability to provide a sensate flap
(E) Lack of abundant soft tissue
The correct response is Option A.
In soft-tissue reconstruction with a superior gluteal artery perforator (SGAP) free flap, the firmer consistency of gluteal fat causes difficulty in molding tissue, particularly in breast reconstruction. Secondary procedures may be necessary to obtain the desired shape.
Gait dysfunction is extremely uncommon after a SGAP flap if care is taken to avoid injury to the motor branch. The scar is generally well hidden by clothing and the donor site deformities are generally minimal. Even in thin patients there is an abundance of adipose tissue in the gluteal area. Although not commonly done, a sensate branch of the nervi clunium superiores can provide sensation to the flap.
Which of the following is a risk factor for hormone-sensitive breast cancer?
A) Breast-feeding
B) Early age at first pregnancy
C) Early menopause
D) Late menarche
E) Post-menopausal obesity
The correct response is Option E.
Post-menopausal obesity is associated with increased adipose production of estrogen, which can increase the risk for hormone-sensitive breast cancer. Other options (late menarche, early menopause, and breast feeding) decrease the number of menstrual cycles, and therefore may decrease the risk of breast cancer. Early age at first pregnancy is also associated with decreased risk for hormone-sensitive breast cancers.
An otherwise healthy 38-year-old woman who is a smoker is considering implant-based breast reconstruction following mastectomy. She has been counseled about likely use of acellular dermal matrix when the intermediate tissue expander is placed and wants to further understand why the matrix will be used in her body. The patient should be advised that use of acellular dermal matrix is associated with a decreased risk for which of the following?
A) Cancer recurrence
B) Capsular contracture
C) Seroma
D) Skin flap necrosis
E) Smoking-related complications
The correct response is Option B.
Acellular dermal matrix (ADM) use has been increasing over time with tissue expander or implant-based breast reconstruction following mastectomies. Many potential advantages and disadvantages have been studied, and some of the data are contradictory. However, the consensus of the literature indicates that ADMs are associated with decreased capsular contracture rates. There is literature to suggest that seroma rates are increased or remain stable, not decreased, with ADMs. ADMs have not been shown to decrease independent patient risk factors for complications such as tobacco use or to decrease cancer recurrence rates. ADMs also do not appear to improve vascularity of the tissue overlying them when initially placed.
A 56-year-old woman presents with right breast swelling 10 years after undergoing bilateral mastectomy and reconstruction with bilateral submuscular textured implants with an acellular dermal matrix sling. Examination shows right-sided periprosthetic fluid collection. Breast implant-associated anaplastic large cell lymphoma is suspected. Analysis of the fluid will most likely show which of the following tumor markers?
A) Anaplastic lymphoma kinase
B) CD3
C) CD5
D) CD30
E) Human germinal center-associated lymphoma (HGAL)
The correct response is Option D.
Breast implant-associated anaplastic large cell lymphoma (BIA-ALCL) has been associated primarily with textured implants. Thought to be related to chronic inflammation from subclinical infection, the presentation is typically with a late-onset seroma. Workup of a late-onset seroma includes aspiration of the seroma and cytological/immunohistochemical analysis. The predictive value is most significant on the initial aspiration because of the presumed larger concentration of malignant cells; as the seroma reforms, there is a hypothesized dilutional aspect.
BIA-ALCL neoplastic cells are strongly CD30-positive, but CD30 is a necessary but insufficient criterion to make the diagnosis since it can be displayed in nonmalignant situations as well (such as in viral infection).
While anaplastic lymphoma kinase (ALK) is associated with systemic ALCL in 60% of cases, it has not been associated with BIA-ALCL. Thus, while ALK assessment is considered necessary to the analysis, it is expected to be negative. BIA-ALCL neoplastic cells variably lose expression of CD3 and CD5. Compared with other forms of ALCL, BIA-ALCL is associated with a more indolent course, although a subset of patients do have an aggressive course. Human germinal center-associated lymphoma (HGAL) is associated with B-cell lymphoma.
An otherwise healthy 45-year-old woman undergoes bilateral breast reconstruction with free deep inferior epigastric perforator-based flaps. Only the medial deep inferior epigastric vein was anastomosed to the medial internal mammary vein using a 3.5-mm venous coupler. Her right breast flap has become increasingly congested. The cutaneous Doppler signals are strong, and examination of the pedicle shows good flow through both arterial and venous anastomoses. There is no kinking or hematoma. Which of the following is the most appropriate next step in management?
A) Additional anastomosis of the lateral deep epigastric vein
B) Additional anastomosis of the superficial inferior epigastric vein
C) Revise the arterial anastomosis
D) Revise both anastomoses
E) Revise the venous anastomosis
The correct response is Option B.
The most appropriate next step in management is to supercharge the flap using an additional anastomosis of the superficial inferior epigastric vein. This is a clinical example of persistent superficial venous system dominance. This is due to either the superficial system being the dominant venous drainage of the abdominal wall and the absence of connections between the superficial and deep venous systems, or the lack of an adequate number of perforators in the flap. In either case, the venous blood is not getting from the superficial system into the deep system, which is the egress given that the deep system is the only system anastomosed.
There is no technical issue with the anastomosis or venous thrombosis; therefore, revising the deep anastomoses is not required. Rather, it is an intra-flap venous system issue that requires the superficial system to be drained somehow. This is accomplished by anastomosing the superficial inferior epigastric vein to either the deep system in the flap or to another drainage system in the chest like the intercostal system. It is interesting to note that the two reasons for the above issue (i.e., inadequate perforator selection or anatomic lack of communicating vessels between the superficial and deep system), though distinct entities, clinically present similarly and are treated in the same way. The incidence of venous congestion secondary to persistent superficial system dominance is about 0.9%.
Which of the following is the most accurate location of the elliptical skin island of a profunda artery perforator (PAP) flap?
A) Anteromedial thigh with the superior border within the groin crease
B) Inferior buttock with the inferior border within the gluteal fold
C) Lateral hip superior to the iliac crest
D) Middle buttock, from the posterior superior iliac spine to the apex of the greater trochanter
E) Posteromedial thigh with the superior border within the gluteal fold
The correct response is Option E.
According to the literature, the skin island of the profunda artery perforator (PAP) flap is inferior to the gluteal fold.1,2 An ellipse of the inferior buttock with the inferior border within the gluteal fold describes the skin island of the inferior gluteal artery perforator (IGAP) free flap.3 An ellipse of the anteromedial thigh with the superior border within the gluteal fold describes the transverse upper gracilis (TUG) flap. An ellipse of the middle buttock, from the posterior superior iliac spine to the apex of the greater trochanter, describes the superior gluteal artery perforator (SGAP) flap.3 An ellipse of the lateral hip superior to the iliac crest describes the Rubens or lateral hip flap.4 The only option that correctly identifies the skin island for the PAP flap is an ellipse of the posteromedial thigh with the superior border within the gluteal fold. The superior marking is within or just below the gluteal fold and the inferior marking is roughly 7 cm below the superior marking. The flap is an ellipse so the scar does not extend outside of the gluteal fold.1,2
A 56-year-old woman with recurrent cancer of the right breast and no evidence of distant metastatic disease is scheduled to undergo completion mastectomy five years after undergoing lumpectomy (segmental mastectomy) for stage II carcinoma with radiation. In this patient, risk of complications is highest with which of the following methods of immediate breast reconstruction?
(A) Free superior gluteal artery perforator (SGAP) flap
(B) Free TRAM flap
(C) Latissimus dorsi myocutaneous flap with implant
(D) Pedicled TRAM flap
(E) Tissue expansion
The correct response is Option E.
In a patient with a previously irradiated breast, reconstruction with a tissue expander followed by a permanent implant (expander/implant) has a higher rate of complications than does reconstruction with any autologous tissue.
The latissimus dorsi myocutaneous flap, pedicled TRAM flap, free TRAM flap, and free SGAP flap are autologous methods of breast reconstruction that bring in new unradiated tissue to reconstruct the breast.
One study showed that previous radiation exposure was a significant risk factor for major complications after breast reconstruction with an expander/implant but not after reconstruction with a TRAM flap. Another study showed that breast cancer patients who underwent radiation and reconstruction with an expander/implant had a significantly higher rate of complications than patients who had radiation and breast reconstruction with a TRAM flap. These findings remained the same whether radiation exposure occurred before or after breast reconstruction.
When performing immediate breast reconstruction, it is important to reconstruct the lateral inframammary fold. This is because the oncologic extirpation of the breast is carried out to which of the following anatomic landmarks?
A) Anterior edge of the latissimus dorsi muscle
B) Anterior edge of the serratus muscle
C) Lateral edge of the pectoralis major muscle
D) Lateral edge of the pectoralis minor muscle
E) Posterior edge of the serratus muscle
The correct response is Option A.
For modified radical and simple mastectomies, the landmarks of dissection are: superiorly to the clavicle, medially to the sternum, inferiorly to the inframammary fold, and laterally to the border of the latissimus dorsi muscle. The pectoralis major muscle fascia is resected with the specimen.
The recreation of the inframammary fold is important for shaping in breast reconstruction and care must be taken to evaluate and repair both the inferior and lateral components of the inframammary fold.
An otherwise healthy 37-year-old woman presents for delayed microsurgical breast reconstruction. Which of the following is associated with use of tamoxifen?
A) Hemodynamic instability
B) Impaired wound healing
C) Increased bleeding
D) Seroma formation
E) Thromboembolic events
The correct response is Option E.
Breast cancers that are estrogen receptor positive may be responsive to adjuvant chemotherapy with selective estrogen receptor modulators such as tamoxifen, which can reduce recurrence and mortality. Tamoxifen is associated with thromboembolic events, such as deep venous thrombosis and pulmonary embolism. This prothrombotic effect has been postulated to be secondary to the effect of tamoxifen on estrogen receptors that are abundant within vascular endothelium.
Tamoxifen has been shown to be associated with increased rates of total flap loss and decreased rates of flap salvage when taken within 28 days of microsurgical breast reconstruction, which represents two half-lives of the active metabolite of tamoxifen (N-desmethyl tamoxifen, t1/2=14 days). It has therefore been recommended that in patients undergoing microsurgical breast reconstruction, tamoxifen be held for at least 28 days preoperatively. Some authors have further advised holding the medication postoperatively in addition to preoperatively.
Tamoxifen is not associated with impaired wound healing, increased bleeding, hemodynamic instability, or seroma formation.
A 44-year-old previously healthy woman comes to the clinic because of a 2-week history of a painless mass in the left breast. She initially felt this mass while taking a shower. Her mother was diagnosed with fibrocystic changes. The patient denies alcohol consumption and smoking cigarettes. Examination of the left breast shows a 5-cm mobile, painless mass in the left upper external quadrant without nipple discharge, skin retractions, or color changes. Examination of a specimen obtained on biopsy discloses a phyllodes tumor, and surgical excision of the lesion is planned. Which of the following is the most important factor to prevent local recurrence after surgery?
A) Adjuvant radiotherapy
B) Concurrent axillary node dissection
C) Postoperative chemotherapy
D) Surgical margins less than or equal to 0.5 cm
E) Wide surgical margins
The correct response is Option E.
In a young woman who has no history of breast cancer, presents with a painless mass, and has a mammogram suggestive of fibroadenoma but a core needle biopsy showing stromal hypercellularity with atypical spindle cells and a high mitotic rate, a phyllodes tumor must be suspected.
Phyllodes tumors are uncommon fibroepithelial breast tumors that behave like benign fibroadenomas, although they have a high propensity to recur locally. More aggressive tumors can metastasize distantly. Surgery is the preferred treatment for this condition. In this context, surgical margins greater than or equal to 1 cm have been associated with a lower recurrence rate in borderline and malignant tumors.
Axillary lymph node involvement is rare. Wide local excision or mastectomy with appropriate margins is the preferred clinical intervention.
Based on limited data, the role of systemic chemotherapy in phyllodes tumors is limited. Patients with benign or borderline phyllodes tumors are usually cured with surgery and should not be offered chemotherapy unless they develop unresectable metastases.
Local recurrence rate is higher after excision with narrower margins than broader ones. The efficacy of postoperative adjuvant radiotherapy for a breast phyllodes tumor is not clear. In clinical practice, the utilization of adjuvant radiotherapy for a phyllodes tumor appears to be modest.
A 45-year-old woman with T3 N0 invasive ductal carcinoma in the inferior pole of the left breast is scheduled to undergo segmental mastectomy and subsequent radiation therapy. She currently wears a size 36 DDD brassiere and is willing to accept any cup size from C to DDD. Which of the following interventions will yield the best cosmetic result in this patient?
A) Bilateral reduction mammaplasty
B) Implantation of a prosthesis in the left breast and mastopexy of the right breast
C) Latissimus dorsi musculocutaneous flap reconstruction of the left breast and mastopexy of the right breast
D) TRAM reconstruction of the defect
E) No reconstruction
Correct answer is option A.
In a patient with large breasts, in whom a partial mastectomy is required, reduction mammaplasty is a very good option. This procedure will potentially relieve symptoms of macromastia, reduce the amount of breast tissue present in both breasts, and offer the best aesthetic outcome.
Implantation of a prosthesis in a breast that will undergo radiation therapy increases the risk for complications and would likely lead to a less symmetrical result.
The latissimus dorsi (or TRAM) flap could be used for immediate partial reconstruction but is ideal for delayed reconstruction of partial mastectomy defects. Reduction mammaplasty does not preclude future reconstruction options, but a latissimus flap reconstruction would.
With a T3 tumor, a significant portion of the lower pole of the breast is removed to obtain clear margins. Even though the patient described has moderate to large breasts, there is a high likelihood that she will develop a deformity of the left breast and asymmetry with the right breast if no reconstruction is performed.